6
$\begingroup$

In a calculation of some momenta of random matrices (GOE), I encounter a functional equation, in the form of a second-order recursion, $$L(s+1)=L(s)+2s(2s+1)L(s-1).$$ Is it familiar to someone ? Is there an explicit formula for the solutions ? By explicit, I mean in terms of the Gamma function for instance.

Just in case, I am interested in the solution generated by $L(0)=1$ and $L(1)=-\frac18$ .

$\endgroup$
3
  • $\begingroup$ @ChristianRemling Oups ! I fix it. Actually, it is $L(s-1)$ instead of $L(s+1)$ in the RHS. $\endgroup$ Commented Feb 17, 2024 at 16:59
  • $\begingroup$ This should succumb pretty easily to generating functions; you can get the $2s(2s+1)L(s-1)$ term by doing the usual differentiation (see e.g. section 2.4 of math.cmu.edu/~ploh/docs/math/2011-228/… ). This'll give you a second-order ODE for $\mathcal{L}(x)=\sum_nL(n)x^n$ that you may have more luck finding. $\endgroup$ Commented Feb 17, 2024 at 18:00
  • 3
    $\begingroup$ FWIW Mathematica gives $$L(s)=2^{s-3}\left(\left(\frac{17}{\sqrt{2}}-9\right)\Gamma\left(s+1\right)+\frac{17(-1)^s}{\sqrt{\pi}(s+1)}\Gamma\left(s+\frac{3}{2}\right){}_2F_1\left(1,s+\frac{3}{2};s+2;-1\right)\right).$$ $\endgroup$ Commented Feb 17, 2024 at 22:55

1 Answer 1

6
$\begingroup$

As already pointed out by @KStarGamer in the comment above, Mathematica can sum the recursion using RSolve[]. The result can be rewritten in terms of the regularized incomplete Euler beta function $I_x(a,b)$ and reads, for arbitrary $L_0=L(0)$ and $L_1=L(1)$, \begin{align} \tag{1a}\label{eq:1a} L(s) &= 2^s s!\left[\frac{2 L_0-L_1}{\sqrt{2}} \big(1 - I_{-1}\left(s+1,\tfrac{1}{2}\right)\big) -L_0 + L_1\right] \\ \tag{1b}\label{eq:1b} &= 2^s s!\left[\frac{2 L_0-L_1}{\sqrt{2}} I_{2}\left(\tfrac{1}{2},s+1\right) -L_0 + L_1\right] \end{align} According to Wikipedia,

The regularized incomplete beta function is the cumulative distribution function of the beta distribution, and is related to the cumulative distribution function $F(k;n,p)$ of a random variable $X$ following a binomial distribution with probability of single success $p$ and number of Bernoulli trials $n$: $$F(k;\,n,p)=\Pr \left(X\leq k\right)=I_{1-p}(n-k,k+1)=1-I_{p}(k+1,n-k).$$

This might link to the OP's original problem.

$\endgroup$
1
  • 1
    $\begingroup$ I apologize that I never learnt softwares such a Mathematica. I should have, but I grew up in the prehistoric times (without personal computers). Thanks a lot ! $\endgroup$ Commented Feb 20, 2024 at 20:01

You must log in to answer this question.

Start asking to get answers

Find the answer to your question by asking.

Ask question

Explore related questions

See similar questions with these tags.